Is g measurable?











up vote
0
down vote

favorite












Let $(X, A)$ be a measurable space and let $f : X to mathbb R$ be a measurable function.



Let $g(x) = 0$ if $f(x)$ is rational



and $g(x) = 1$ if $f(x)$ is irrational.



Am I correct in saying $g$ is measurable ?










share|cite|improve this question
























  • You can write $h(a) = mathbb 1(atext{ is rational})$, then $g(x)=h(f(x))$, $f$ is measurable and clearly $h$ is measurable too, so $hcirc f$ is measurable. Of course you need to justify that $h$ is measurable.
    – P. Quinton
    Nov 18 at 12:16










  • How would you prove h to be measurable I know that for h to be measurable it must be a function between two measurable spaces such that the preimage of any measurable set is measurable . @P.Quinton
    – Johnmallu
    Nov 18 at 12:20












  • Observe that any subset of $mathbb Q$ is measurable, hence so is $mathbb Rsetminus mathbb Q$. For any measurable set $Xsubseteq mathbb R$, the two sets $Xcap mathbb Q$ and $xsetminus mathbb Q$ are also measurable and the image of $X$ with respect to $h$ is directly a function of those two sets.
    – P. Quinton
    Nov 18 at 12:32












  • Ah I see. Thank you for clarifying and the explanation.
    – Johnmallu
    Nov 18 at 12:39















up vote
0
down vote

favorite












Let $(X, A)$ be a measurable space and let $f : X to mathbb R$ be a measurable function.



Let $g(x) = 0$ if $f(x)$ is rational



and $g(x) = 1$ if $f(x)$ is irrational.



Am I correct in saying $g$ is measurable ?










share|cite|improve this question
























  • You can write $h(a) = mathbb 1(atext{ is rational})$, then $g(x)=h(f(x))$, $f$ is measurable and clearly $h$ is measurable too, so $hcirc f$ is measurable. Of course you need to justify that $h$ is measurable.
    – P. Quinton
    Nov 18 at 12:16










  • How would you prove h to be measurable I know that for h to be measurable it must be a function between two measurable spaces such that the preimage of any measurable set is measurable . @P.Quinton
    – Johnmallu
    Nov 18 at 12:20












  • Observe that any subset of $mathbb Q$ is measurable, hence so is $mathbb Rsetminus mathbb Q$. For any measurable set $Xsubseteq mathbb R$, the two sets $Xcap mathbb Q$ and $xsetminus mathbb Q$ are also measurable and the image of $X$ with respect to $h$ is directly a function of those two sets.
    – P. Quinton
    Nov 18 at 12:32












  • Ah I see. Thank you for clarifying and the explanation.
    – Johnmallu
    Nov 18 at 12:39













up vote
0
down vote

favorite









up vote
0
down vote

favorite











Let $(X, A)$ be a measurable space and let $f : X to mathbb R$ be a measurable function.



Let $g(x) = 0$ if $f(x)$ is rational



and $g(x) = 1$ if $f(x)$ is irrational.



Am I correct in saying $g$ is measurable ?










share|cite|improve this question















Let $(X, A)$ be a measurable space and let $f : X to mathbb R$ be a measurable function.



Let $g(x) = 0$ if $f(x)$ is rational



and $g(x) = 1$ if $f(x)$ is irrational.



Am I correct in saying $g$ is measurable ?







measure-theory






share|cite|improve this question















share|cite|improve this question













share|cite|improve this question




share|cite|improve this question








edited Nov 18 at 13:06









amWhy

191k27223439




191k27223439










asked Nov 18 at 12:09









Johnmallu

266




266












  • You can write $h(a) = mathbb 1(atext{ is rational})$, then $g(x)=h(f(x))$, $f$ is measurable and clearly $h$ is measurable too, so $hcirc f$ is measurable. Of course you need to justify that $h$ is measurable.
    – P. Quinton
    Nov 18 at 12:16










  • How would you prove h to be measurable I know that for h to be measurable it must be a function between two measurable spaces such that the preimage of any measurable set is measurable . @P.Quinton
    – Johnmallu
    Nov 18 at 12:20












  • Observe that any subset of $mathbb Q$ is measurable, hence so is $mathbb Rsetminus mathbb Q$. For any measurable set $Xsubseteq mathbb R$, the two sets $Xcap mathbb Q$ and $xsetminus mathbb Q$ are also measurable and the image of $X$ with respect to $h$ is directly a function of those two sets.
    – P. Quinton
    Nov 18 at 12:32












  • Ah I see. Thank you for clarifying and the explanation.
    – Johnmallu
    Nov 18 at 12:39


















  • You can write $h(a) = mathbb 1(atext{ is rational})$, then $g(x)=h(f(x))$, $f$ is measurable and clearly $h$ is measurable too, so $hcirc f$ is measurable. Of course you need to justify that $h$ is measurable.
    – P. Quinton
    Nov 18 at 12:16










  • How would you prove h to be measurable I know that for h to be measurable it must be a function between two measurable spaces such that the preimage of any measurable set is measurable . @P.Quinton
    – Johnmallu
    Nov 18 at 12:20












  • Observe that any subset of $mathbb Q$ is measurable, hence so is $mathbb Rsetminus mathbb Q$. For any measurable set $Xsubseteq mathbb R$, the two sets $Xcap mathbb Q$ and $xsetminus mathbb Q$ are also measurable and the image of $X$ with respect to $h$ is directly a function of those two sets.
    – P. Quinton
    Nov 18 at 12:32












  • Ah I see. Thank you for clarifying and the explanation.
    – Johnmallu
    Nov 18 at 12:39
















You can write $h(a) = mathbb 1(atext{ is rational})$, then $g(x)=h(f(x))$, $f$ is measurable and clearly $h$ is measurable too, so $hcirc f$ is measurable. Of course you need to justify that $h$ is measurable.
– P. Quinton
Nov 18 at 12:16




You can write $h(a) = mathbb 1(atext{ is rational})$, then $g(x)=h(f(x))$, $f$ is measurable and clearly $h$ is measurable too, so $hcirc f$ is measurable. Of course you need to justify that $h$ is measurable.
– P. Quinton
Nov 18 at 12:16












How would you prove h to be measurable I know that for h to be measurable it must be a function between two measurable spaces such that the preimage of any measurable set is measurable . @P.Quinton
– Johnmallu
Nov 18 at 12:20






How would you prove h to be measurable I know that for h to be measurable it must be a function between two measurable spaces such that the preimage of any measurable set is measurable . @P.Quinton
– Johnmallu
Nov 18 at 12:20














Observe that any subset of $mathbb Q$ is measurable, hence so is $mathbb Rsetminus mathbb Q$. For any measurable set $Xsubseteq mathbb R$, the two sets $Xcap mathbb Q$ and $xsetminus mathbb Q$ are also measurable and the image of $X$ with respect to $h$ is directly a function of those two sets.
– P. Quinton
Nov 18 at 12:32






Observe that any subset of $mathbb Q$ is measurable, hence so is $mathbb Rsetminus mathbb Q$. For any measurable set $Xsubseteq mathbb R$, the two sets $Xcap mathbb Q$ and $xsetminus mathbb Q$ are also measurable and the image of $X$ with respect to $h$ is directly a function of those two sets.
– P. Quinton
Nov 18 at 12:32














Ah I see. Thank you for clarifying and the explanation.
– Johnmallu
Nov 18 at 12:39




Ah I see. Thank you for clarifying and the explanation.
– Johnmallu
Nov 18 at 12:39










1 Answer
1






active

oldest

votes

















up vote
1
down vote



accepted










The inverse image of any Borel set under $g$ is $emptyset$, $X$, $f^{-1}(mathbb Q)$ or $f^{-1}(mathbb Q^{c})$ and each of these sets is measurable.






share|cite|improve this answer





















    Your Answer





    StackExchange.ifUsing("editor", function () {
    return StackExchange.using("mathjaxEditing", function () {
    StackExchange.MarkdownEditor.creationCallbacks.add(function (editor, postfix) {
    StackExchange.mathjaxEditing.prepareWmdForMathJax(editor, postfix, [["$", "$"], ["\\(","\\)"]]);
    });
    });
    }, "mathjax-editing");

    StackExchange.ready(function() {
    var channelOptions = {
    tags: "".split(" "),
    id: "69"
    };
    initTagRenderer("".split(" "), "".split(" "), channelOptions);

    StackExchange.using("externalEditor", function() {
    // Have to fire editor after snippets, if snippets enabled
    if (StackExchange.settings.snippets.snippetsEnabled) {
    StackExchange.using("snippets", function() {
    createEditor();
    });
    }
    else {
    createEditor();
    }
    });

    function createEditor() {
    StackExchange.prepareEditor({
    heartbeatType: 'answer',
    convertImagesToLinks: true,
    noModals: true,
    showLowRepImageUploadWarning: true,
    reputationToPostImages: 10,
    bindNavPrevention: true,
    postfix: "",
    imageUploader: {
    brandingHtml: "Powered by u003ca class="icon-imgur-white" href="https://imgur.com/"u003eu003c/au003e",
    contentPolicyHtml: "User contributions licensed under u003ca href="https://creativecommons.org/licenses/by-sa/3.0/"u003ecc by-sa 3.0 with attribution requiredu003c/au003e u003ca href="https://stackoverflow.com/legal/content-policy"u003e(content policy)u003c/au003e",
    allowUrls: true
    },
    noCode: true, onDemand: true,
    discardSelector: ".discard-answer"
    ,immediatelyShowMarkdownHelp:true
    });


    }
    });














    draft saved

    draft discarded


















    StackExchange.ready(
    function () {
    StackExchange.openid.initPostLogin('.new-post-login', 'https%3a%2f%2fmath.stackexchange.com%2fquestions%2f3003454%2fis-g-measurable%23new-answer', 'question_page');
    }
    );

    Post as a guest















    Required, but never shown

























    1 Answer
    1






    active

    oldest

    votes








    1 Answer
    1






    active

    oldest

    votes









    active

    oldest

    votes






    active

    oldest

    votes








    up vote
    1
    down vote



    accepted










    The inverse image of any Borel set under $g$ is $emptyset$, $X$, $f^{-1}(mathbb Q)$ or $f^{-1}(mathbb Q^{c})$ and each of these sets is measurable.






    share|cite|improve this answer

























      up vote
      1
      down vote



      accepted










      The inverse image of any Borel set under $g$ is $emptyset$, $X$, $f^{-1}(mathbb Q)$ or $f^{-1}(mathbb Q^{c})$ and each of these sets is measurable.






      share|cite|improve this answer























        up vote
        1
        down vote



        accepted







        up vote
        1
        down vote



        accepted






        The inverse image of any Borel set under $g$ is $emptyset$, $X$, $f^{-1}(mathbb Q)$ or $f^{-1}(mathbb Q^{c})$ and each of these sets is measurable.






        share|cite|improve this answer












        The inverse image of any Borel set under $g$ is $emptyset$, $X$, $f^{-1}(mathbb Q)$ or $f^{-1}(mathbb Q^{c})$ and each of these sets is measurable.







        share|cite|improve this answer












        share|cite|improve this answer



        share|cite|improve this answer










        answered Nov 18 at 12:32









        Kavi Rama Murthy

        44.1k31852




        44.1k31852






























            draft saved

            draft discarded




















































            Thanks for contributing an answer to Mathematics Stack Exchange!


            • Please be sure to answer the question. Provide details and share your research!

            But avoid



            • Asking for help, clarification, or responding to other answers.

            • Making statements based on opinion; back them up with references or personal experience.


            Use MathJax to format equations. MathJax reference.


            To learn more, see our tips on writing great answers.





            Some of your past answers have not been well-received, and you're in danger of being blocked from answering.


            Please pay close attention to the following guidance:


            • Please be sure to answer the question. Provide details and share your research!

            But avoid



            • Asking for help, clarification, or responding to other answers.

            • Making statements based on opinion; back them up with references or personal experience.


            To learn more, see our tips on writing great answers.




            draft saved


            draft discarded














            StackExchange.ready(
            function () {
            StackExchange.openid.initPostLogin('.new-post-login', 'https%3a%2f%2fmath.stackexchange.com%2fquestions%2f3003454%2fis-g-measurable%23new-answer', 'question_page');
            }
            );

            Post as a guest















            Required, but never shown





















































            Required, but never shown














            Required, but never shown












            Required, but never shown







            Required, but never shown

































            Required, but never shown














            Required, but never shown












            Required, but never shown







            Required, but never shown







            Popular posts from this blog

            QoS: MAC-Priority for clients behind a repeater

            Ивакино (Тотемский район)

            Can't locate Autom4te/ChannelDefs.pm in @INC (when it definitely is there)